LSAT and Law School Admissions Forum

Get expert LSAT preparation and law school admissions advice from PowerScore Test Preparation.

 Administrator
PowerScore Staff
  • PowerScore Staff
  • Posts: 8916
  • Joined: Feb 02, 2011
|
#40632
Complete Question Explanation
(The complete setup for this game can be found here: lsat/viewtopic.php?t=14136)

The correct answer choice is (A)

In this List question, the rules should be applied in the following order: second, third, first, and fourth (although note that the second and third rules are equally easy to apply, and the first and fourth rules are similar in application difficulty as well).

Answer choice (A): This is the correct answer choice.

Answer choice (B): This answer choice violates the second rule and is therefore incorrect.

Answer choice (C): This answer choice violates the first rule and is therefore incorrect.

Answer choice (D): This answer choice violates the third rule and is therefore incorrect.

Answer choice (E): This answer choice violates the fourth rule and is therefore incorrect.

As is often the case in List questions, each of the incorrect answers violates a different rule.

Get the most out of your LSAT Prep Plus subscription.

Analyze and track your performance with our Testing and Analytics Package.